Đến nội dung

nmlinh16 nội dung

Có 149 mục bởi nmlinh16 (Tìm giới hạn từ 08-05-2020)



Sắp theo                Sắp xếp  

#742557 Một số tự nhiên được gọi là số “đẹp” nếu nó có thể phân tích được thành tích...

Đã gửi bởi nmlinh16 on 18-12-2023 - 03:16 trong Số học

Một cách để đoán ra được đáp án này 

 

Vậy tập hợp các số $x_i$ chỉ bao gồm các số $2$ và $3$, và có không quá hai số $2$.

là giải bài toán liên tục trước.

 

Đầu tiên, ta tìm các số thực dương $x_1,\ldots,x_n$ với tông $x_1 + \cdots + x_n = a$ không đổi (ở đây là $a = 2020$), sao cho tích $x_1 \cdots x_n$. Bằng bất đẳng thức AM-GM, ta thấy điều này đạt được khi $x_1 = \cdots = x_n = \frac{a}{n}$, và khi đó $x_1 \cdots x_n = \left(\frac{a}{n}\right)^n$.

Ta liên tục hóa thêm một lần nữa: tìm số thực $x > 0$ sao cho hàm $f(x) = \left(\frac{a}{x}\right)^x$ đạt giá trị lớn nhất. Đặt $g(x) = \ln (f(x))$ và khảo sát hàm số $g$, ta thấy $g$ đạt giá trị lớn nhất khi $x = \frac{a}{e}$.

Vậy ta phán đoán rằng tích $x_1 \cdots x_n$ đạt giá trị lớn nhất khi $x_1,\ldots,x_n$ "gần nhau nhất có thể" (hay đôi một hơn kém nhau không quá $1$ đơn vị), và chúng đều gần $e$ nhất có thể (hay nhận giá trị $2$ và $3$).

 

Khi đã đoán ra được đáp án trên thì ta chứng minh như @perfectstrong.




#742553 Trên $x,y \in C^{1}[0,1]$, cho M là tập hợp các đa t...

Đã gửi bởi nmlinh16 on 17-12-2023 - 23:14 trong Giải tích

Một cơ sở cho $M$ là $\{t, t^2, t^3\}$. Tính các tích vô hướng $\langle{t^i,t^j\rangle}$ với $1 \le i,j \le 3$, rồi áp dụng phép trực giao hóa Gram-Schmidt.




#742532 Sự thú vị của con số 1/998001

Đã gửi bởi nmlinh16 on 16-12-2023 - 16:26 trong Toán học lý thú

@hxthanh tính đúng rồi.

Kết luận này của em

 

(chú ý là nó không phải số thập phân vô hạn tuần hoàn $0.(000001002\ldots 997999)$.

Bằng cách tính các tổng tương tự như vậy (thay $x = 1/10^k$, với $k \in \mathbb{N}^\ast$), ta có thể tìm được các số tương tư $998001$.

là sai.




#742521 Chứng minh ánh xạ song tuyến tính, liên tục theo từng biến nhưng không liên t...

Đã gửi bởi nmlinh16 on 16-12-2023 - 01:52 trong Giải tích

$B$ liên tục theo biến $p$ khi cố định $q$:

Vì $q$ là hàm đa thức nên liên tục, vì thế bị chặn trên $[0,1]$, hay tồn tại $C > 0$ để $|q(t)| \le C$ với mọi $t \in [0,1]$, từ đó $$|B(p,q)| \le \int_0^1 |p(t)|C\,dt = C\|p\|$$ với mọi $p \in X$, hay ánh xạ tuyến tính $p \mapsto B(p,q)$ liên tục.

$B$ không là ánh xạ song tuyến tính liên tục:

Với mọi $n$ nguyên dương, xét $p_n(t) = t^n$, $q_n(t) = t^n$. Thế thì $$\|p_n\| = \|q_n\| = \int_0^1 t^n\,dt = \frac{1}{n+1}.$$ Mà $$B(p_n,q_n) = \int_0^1 t^{2n}\,dt = \frac{1}{2n+1},$$ suy ra $$\lim_{n \to +\infty} \frac{|B(p_n,q_n)|}{\|p_n\|  \|q_n\|} = \lim_{n \to +\infty}\frac{(n+1)^2}{2n+1}  = +\infty,$$ vì thế không tồn tại hằng số $C > 0$ sao cho $$|B(p,q)| \le C \|p\| \|q\|$$ với mọi $p,q \in X$, nên $B$ không là ánh xạ song tuyến tính liên tục.




#742520 Sự thú vị của con số 1/998001

Đã gửi bởi nmlinh16 on 16-12-2023 - 01:31 trong Toán học lý thú

Bắt đầu từ đẳng thức $$\frac{1}{1-x} = \sum_{i=0}^\infty x^i,$$ ta đạo hàm hai vế, rồi nhân hai vế với $x^2$ để thu được $$\frac{x^2}{(1-x)^2} = \sum_{i=0}^\infty ix^{i+1}.$$ Thay $x = \frac{1}{1000}$, ta được $$\frac{1}{998001} = \sum_{i=0}^\infty ix^{i+1} = \frac{0}{1000} + \frac{1}{1000^2} + \frac{2}{1000^3} + \cdots + \frac{997}{1000^{998}} + \frac{998}{1000^{999}} + \frac{999}{1000^{1000}} + \frac{1000}{1000^{1001}} + \sum_{i=1001}^{\infty} \frac{i}{1000^{i+1}}.$$ Bây giờ, nhân xét rằng $$\begin{align*} \frac{998}{1000^{999}} + \frac{999}{1000^{1000}} + \frac{1000}{1000^{1001}} & = \frac{998}{1000^{999}} + \frac{999}{1000^{1000}} + \frac{1}{1000^{1000}} \\ & =  \frac{998}{1000^{999}} + \frac{1000}{1000^{1000}} \\ & = \frac{998}{1000^{999}} + \frac{1}{1000^{999}} \\ & = \frac{999}{1000^{999}}. \end{align*}$$ Vậy ta có $$\frac{1}{998001} =  \frac{0}{1000} + \frac{1}{1000^2} + \frac{2}{1000^3} + \cdots + \frac{997}{1000^{998}} + \frac{999}{1000^{999}} + \sum_{i=1001}^{\infty} \frac{i}{1000^{i+1}}.$$ Nhận xét thêm rằng với $n \in \mathbb{N}^\ast$ thì $$\frac{x^{n+1} - 1}{x-1} = \sum_{i=0}^n x^i.$$ Đạo hàm hai vế rồi nhân với $x^2$, ta được $$\frac{x^2 (n x^{n+1} - (n+1)x^n + 1)}{(x-1)^2} = \sum_{i=0}^n ix^{i+1}.$$ Từ đó ta có $$\sum_{i=n+1}^\infty ix^{i+1} = \sum_{i=0}^\infty ix^{i+1} - \sum_{i=0}^n ix^{i+1} = \frac{((n+1)x^n - nx^{n+1})x^2}{(x-1)^2} = \frac{((n+1) - nx)x^{n+2}}{(x-1)^2}.$$ Thay $x = \frac{1}{1000}$ và $n = 1000$, ta thu được $$\sum_{i=1001}^{\infty} \frac{i}{1000^{i+1}} = \frac{1000 \cdot 1000^2}{999^2 \cdot 1000^{1002}} < \frac{1}{1000^{1000}}.$$ Vậy $$ \begin{align*} & \frac{0}{1000} + \frac{1}{1000^2} + \frac{2}{1000^3} + \cdots + \frac{997}{1000^{998}} + \frac{999}{1000^{999}} \\ <\ & \frac{1}{998001} \\ <\ & \frac{0}{1000} + \frac{1}{1000^2} + \frac{2}{1000^3} + \cdots + \frac{997}{1000^{998}} + \frac{999}{1000^{999}} + \frac{1}{1000^{1000}}, \end{align*}$$ điều này giải thích vì sao $$\frac{1}{998001}  = 0.000001002\ldots 997999 \ldots $$ (chú ý là nó không phải số thập phân vô hạn tuần hoàn $0.(000001002\ldots 997999)$.

Bằng cách tính các tổng tương tự như vậy (thay $x = 1/10^k$, với $k \in \mathbb{N}^\ast$), ta có thể tìm được các số tương tư $998001$.




#742466 Xét sự hội tụ của dãy $(x_{n})$ xác định bởi $x_...

Đã gửi bởi nmlinh16 on 11-12-2023 - 01:03 trong Giải tích

Ta chứng minh $(x_n)$ không hội tụ theo chuẩn $|| \cdot ||$.

Thật vậy, sự hội tụ theo chuẩn $|| \cdot ||$ chính là hội tụ đều, nên nếu $x_n$ hội tụ về một hàm $x$ theo chuẩn $|| \cdot ||$ thì $x_n$ hội tụ đều về $x$, nói riêng thì nó hội tụ điểm, hay $\lim_{n \to +\infty} x_n(t) = x(t)$ với mọi $t \in [0,1]$.

Nhận xét rằng, với $0 \le t < 1$ thì $\lim_{n \to +\infty} nt^n = 0$, nên $\lim_{n \to +\infty} x_n(t) = t^3$. 

Với $t = 1$ thì $x_n(1) = 1 = 1^3$ với mọi $n$.

Vậy ta có $\lim_{n \to +\infty} x_n(t) = t^3$ với mọi $t \in [0,1]$. Vậy $x(t) = t^3$ với mọi $t \in [0,1]$.

Từ đây ta sẽ chỉ ra điều mâu thuẫn rằng $x_n$ không hội tụ về $x$ theo chuẩn $||\cdot||$. Thật vậy, ta có $$||x_n - x|| \ge ||x_n(\tfrac{n}{n+1}) - x(\tfrac{n}{n+1})|| = (\tfrac{n}{n+1})^{n+1} \to \tfrac{1}{e}$$ khi $n \to +\infty$. 

Vậy ta kết luận rằng $(x_n)$ không hội tụ theo chuẩn $|| \cdot ||$.

 

Ngược lại, ta kiểm tra rằng $$||x_n - x||_1 = \int_0^1 |nt^n - nt^{n+1}|\,dt = \int_0^1 (nt^n - nt^{n+1})\,dt = \tfrac{n}{n+1} - \tfrac{n}{n+2} \to 0$$ khi $n \to +\infty$, nên $(x_n)$ hội tụ về hàm $x(t) = t^3$, theo chuẩn $||\cdot||_1$.




#742320 $sup\bigcup_{k=1}^{n} \rightarrow sup...

Đã gửi bởi nmlinh16 on 02-12-2023 - 01:11 trong Giải tích

Đặt $a_n:=\sup\left(\bigcup_{k=1}^n A_k\right)$ và $a:=\sup\left(\bigcup_{k=1}^{\infty} A_k\right)$. Dãy $(a_n)_{n \ge 1}$ tăng và bị chặn trên bởi $a$ nên hội tụ. Đặt $\lim_{n \to \infty} a_n =: b$ thì $b \le a$. Ta giả sử $b < a$. Theo định nghĩa cận trên đúng, tồn tại $c \in \bigcup_{k=1}^{\infty} A_k$ sao cho $b < c \le a$. Lấy số nguyên dương $k$ sao cho $c \in A_k$. Thế thì $c \le a_k$ theo định nghĩa của $a_k$, suy ra $c \le b$, mâu thuẫn. Vậy $b = a$ (chính là điều cần chứng minh).




#742161 Ánh xạ transgression (tràn) trong dãy phổ Hochschild-Serre

Đã gửi bởi nmlinh16 on 19-11-2023 - 05:49 trong Toán học hiện đại

Cho $k$ là một trường và ký hiệu $\bar{k}$ là một bao đóng tách được của nó, và $\Gamma:=\text{Gal}(\bar{k}/k)$.

Cho $X$ là một đa tạp trơn và bất khả quy hình học trên $k$. Ký hiệu $\bar{X} = X \times_k \bar{k}$.

Cho $G$ là một nhóm đại số trơn và giao hoán trên $k$.  Ký hiệu $\bar{G} = G \times_k \bar{k}$.

Tác động của $\Gamma$ trên $X(\bar{k})$ và $G(\bar{k})$ sẽ lần lượt được ký hiệu bởi $(s,x) \mapsto {}^s x$ và $(s,g) \mapsto {}^s g$.

 

Các nhóm đối đồng điều dưới đây đều là đối đồng điều Galois hoặc étale.

 

Ta nhắc lại rằng $\Gamma$ tác động lên $H^0(\bar{X},\bar{G})$ và $H^1(\bar{X},\bar{G})$ như sau.

  1. Một phần tử của $H^0(\bar{X},\bar{G}) = \bar{G}(\bar{X})$ là một $\bar{k}$-cấu xạ $\sigma: \bar{X} \to \bar{G}$. Với $s \in \Gamma$, ta định nghĩa ${}^s \sigma: \bar{X} \to \bar{G}$ là cấu xạ cho bởi công thức $x \mapsto {}^s (\sigma({}^{s^{-1}} x))$. 
  2. Một phần tử của $H^1(\bar{X},\bar{G})$ được biểu diễn bởi một torsor $f: \bar{Y} \to \bar{X}$ dưới $\bar{G}$, nghĩa là $f$ là fppf, $\bar{Y}$ được trang bị một tác động phải $(y,g) \mapsto y \cdot g$ của $\bar{G}$ sao cho $f(y \cdot g) = f(y)$, và với mỗi $x \in X(\bar{k})$ thì $G(\bar{k})$ tác động truyền dẫn và tự do trên $f^{-1}(x)$ (xem thêm: https://diendantoanh...về-các-cản-trở/). Với mỗi $s \in \Gamma$, ta định nghĩa torsor liên hợp ${}^s f: {}^s \bar{Y} \to \bar{X}$ như sau: ${}^s\bar{Y} := \bar{Y}$, cấu xạ ${}^s f$ được cho bởi $({}^s f)(y):= {}^s (f(y))$, và tác động của $\bar{G}$ trên ${}^s \bar{Y}$ được cho bởi $(y,g) \mapsto y \cdot {}^{s^{-1}} g$.

Ta ký hiệu $E$ là tập hợp các cặp $(\alpha,s)$, trong đó $s \in \Gamma$ và $\alpha: {}^s \bar{Y} \to \bar{Y}$ là một đẳng cấu (giữa hai $\bar{X}$-torsor dưới $\bar{G}$. Dễ thấy $E$ có một cấu trúc nhóm hiển nhiên. Ngoài ra, nhắc lại rằng nhóm các $\bar{X}$-tự đẳng cấu của torsor $\bar{Y}$ dưới $\bar{G}$  chính là nhóm $\bar{G}(\bar{X})$ (xem Lemma 4.1 trong https://www-fourier....torsors_rev.pdf). Vậy ta có dãy khớp $$1 \to \bar{G}(\bar{X}) \to E \xrightarrow{q} \Gamma,$$ trong đó $q$ là một toàn ánh khi và chỉ khi $[\bar{Y}] \in H^0(k,H^1(\bar{X},\bar{G}))$.

 

Câu hỏi: Ta có dãy khớp 5 hạng tử $$0 \to H^1(k,\bar{G}(\bar{X})) \to H^1(X,G) \to H^0(k,H^1(\bar{X},\bar{G})) \xrightarrow{\partial} H^2(k,\bar{G}(\bar{X})) \to H^2(X,G)$$ rút ra từ dãy phổ Hochschild-Serre $$H^p(k,H^q(\bar{X},\bar{G})) \Rightarrow H^{p+q}(X,G).$$ Các ánh xạ trong dãy khớp trên đều hiển nhiên trừ $\partial$, nó được gọi là ánh xạ tràn (transgression). Chứng minh rằng nếu $\bar{Y} \to \bar{X}$ là một torsor dưới $\bar{G}$ sao cho $[\bar{Y}] \in H^0(k,H^1(\bar{X},\bar{G}))$ thì $\partial([\bar{Y}]) \in H^2(k,\bar{G}(\bar{X}))$ là lớp đối đồng điều được biểu diễn bởi dãy khớp $$1 \to \bar{G}(\bar{X}) \to E \xrightarrow{q} \Gamma \to 1.$$




#742125 CMR: tồn tại duy nhất $(u_{1},..,u_{H})$ và...

Đã gửi bởi nmlinh16 on 14-11-2023 - 17:31 trong Số học

Về câu chuyện tập sinh nhỏ nhất của nhóm $(\mathbb{Z}/n)^\times$, theo hiểu biết của mình thì chưa có công thức tổng quát để tính một cách hiệu quả tập sinh này, nhưng có một số bình luân có thể có ích.

Để theo dõi các thảo luận ở dưới thì cần có một chút kiến thức về lý thuyết nhóm, cụ thể là định lý về cấu trúc của nhóm abel hữu hạn.

 

 

1. Ta giả sử $n$ có phân tích ra thừa số nguyên tố là $n = 2^k \prod_{i=1}^{f} p_i^{k_i}$, trong đó $p_i$ là các số nguyên tố lẻ phân biệt, $k_i > 0$, và $k \ge 0$ (có một chút rắc rói với số nguyên tố 2 như mình đã nói ở post trước, nên ta sẽ tách riêng nó ra).

Theo định lý số dư Trung Hoa, ta có đẳng cấu nhóm $$(\mathbb{Z}/n)^\times \xrightarrow{\cong} (\mathbb{Z}/2^k)^\times \times \prod_{i=1}^{f} (\mathbb{Z}/p_i^{k_i})^\times, \quad a \mod m \mapsto (a \mod 2^k, a \mod p_1^{k_1},\ldots, a \mod p_f^{k_f}),$$

nên ta có thể đưa về nghiên cứu các nhóm $(\mathbb{Z}/n)^\times$ với $n$ là lũy thừa nguyên tố.

 

  • Với $k = 0, 1$ thì $(\mathbb{Z}/2^k)^\times = \{1\}$.
  • Với $k = 2$ thì $(\mathbb{Z}/4)^\times = \{1,3\} \cong \mathbb{Z}/2$, là nhóm cyclic với phần tử sinh là $3$.
  • Với $k \ge 3$ thì không khó để chứng minh rằng $(\mathbb{Z}/2^k)^\times \cong (\mathbb{Z}/2) \times (\mathbb{Z}/2^{k-2})$ là nhóm bicyclic với hai phần tử sinh là $-1$ và $3$.

 

Tiếp theo, với $p$ là số nguyên tố lẻ $(\mathbb{Z}/p)^\times$ luôn là nhóm cyclic (đẳng cấu với $\mathbb{Z}/(p-1)$), tức là được sinh bởi $1$ phần tử. Nói theo ngôn ngữ phổ thông là luôn tồn tại căn nguyên thủy modulo $p$. Nhưng theo hiểu biết của mình thì không có công thức tổng quát để tìm một phần tử sinh của nhóm này.

Sau đó, ta có thể chứng minh rằng nếu $\varepsilon$ là một phần tử sinh của $(\mathbb{Z}/p^k)^\times$ thì ít nhất moojtr trong hai số $\varepsilon$ hoặc $\varepsilon + p$ sẽ là một phần tử sinh của $(\mathbb{Z}/p^{k+1})^\times$. Từ đó ta có thể dùng quy nạp để tính dần dần một phần tử sinh của $(\mathbb{Z}/p^k)^\times$ với $k \ge 1$ và $p$ nguyên tố lẻ. Nói riêng, $(\mathbb{Z}/p^k)^\times \cong \mathbb{Z}/p^{k-1}(p-1)$.

 

Khi tìm được các phần tử sinh $\varepsilon_i$ của $(\mathbb{Z}/p_i^{k_i})^\times$, ta tìm nghiệm nguyên $u_i$ của hệ phương trình đồng dư $$\begin{cases} u_i \equiv \varepsilon_i \pmod{p_i^{k_i}}\\ u_i \equiv 1 \pmod{2^k \prod_{\substack{j=1 \\ j \neq i}}^f p_j^{k_j}}.\end{cases}$$

 

Tóm lại, với phân tích của $n$ ra thừa số nguyên tố như trên,

  • Với $k=0,1$ thì $(\mathbb{Z}/n)^\times \cong \prod_{i=1}^f \mathbb{Z}/p_i^{k_i-1}(p_i-1)$ với $f$ phần tử sinh là $u_1,\ldots,u_f$.
  • Với $k=2$ thì $(\mathbb{Z}/n)^\times \cong (\mathbb{Z}/2) \times \prod_{i=1}^f \mathbb{Z}/p_i^{k_i-1}(p_i-1)$ với $f+1$ phần tử sinh là $u,u_1,\ldots,u_f$, trong đó $u$ là một nghiệm của hệ phương trình đồng dư $$\begin{cases} u \equiv 3 \pmod{2^k} \\ u \equiv 1 \pmod{\prod_{i=1}^{f} p_i^{k_i}}. \end{cases}$$
  • Với $k\ge 3$ thì $(\mathbb{Z}/n)^\times \cong (\mathbb{Z}/2) \times (\mathbb{Z}/2^{k-2}) \times \prod_{i=1}^f \mathbb{Z}/p_i^{k_i-1}(p_i-1)$ với $f+2$ phần tử sinh là $u,u',u_1,\ldots,u_f$, trong đó $u$ như trên và $u'$ là một nghiệm của hệ phương trình đồng dư $$\begin{cases} u \equiv -1 \pmod{2^k} \\ u \equiv 1 \pmod{\prod_{i=1}^{f} p_i^{k_i}}. \end{cases}$$

 

2. Ta sẽ chỉ ra tập sinh như trên là tập sinh (có số phần tử) nhỏ nhất. Để làm điều này ta cần khái niệm nhân tử bất biến cho nhóm abel hữu hạn. Định lý cơ bản về lý thuyết nhóm abel hữu hạn nói rằng mỗi nhóm abel hữu hạn $A$ đều có thể phân tích thành $A \cong (\mathbb{Z}/d_1) \times \cdots \times (\mathbb{Z}/d_\ell)$, trong đó các số nguyên dương $d_1,\ldots,d_\ell$ được xác định một các duy nhất sao cho $d_1 > 1$ và $d_1 | d_2 | \cdots | d_\ell$. Các số nguyên dương $d_i$ được gọi là các nhân tử bất biến của $A$, và $\ell$ được gọi là độ dài bất biến của $A$.

Ví dụ, $(\mathbb{Z}/2) \times (\mathbb{Z}/3) \cong \mathbb{Z}/6$ có độ dài bất biến bằng $1$ và nhân tử bất biến duy nhất là $6$, trong khi $(\mathbb{Z}/4) \times (\mathbb{Z}/10) \cong (\mathbb{Z}/2) \times (\mathbb{Z}/4) \times (\mathbb{Z}/5) \cong (\mathbb{Z}/2) \times (\mathbb{Z}/20)$ có độ dài bất biến bằng $2$ và hai nhân tử bất biến là $2$ và $20$.

 

Ta chỉ ra được rằng độ dài bất biến của $A$ chính là số phần tử của bất kỳ tập sinh nhỏ nhất nào, xem https://math.stackex...nite-abelian-gr

Khi ta có một phân tích $A \cong \prod_{i=1}^r \mathbb{Z}/m_i$ thì $A$ được sinh bởi $r$ phần tử, nhưng $r$ không nhất thiết là độ dài bất biến (tức là số phần tử sinh nhỏ nhất), mà ta chỉ biết rằng độ dài bất biến không vượt quá $r$.

Tuy nhiên, nếu có thểm giả thiết rằng $\gcd(m_1,\ldots,m_r) > 1$ thì ta chứng minh được rằng $r$ chính là độ dài bất biến: xem chứng minh của Lemma 5.1, trang 10 trong https://personal.mat...loads/SIFMG.pdf

Áp dụng cho nhóm $A = (\mathbb{Z}/n)^\times$ như trên (ta thấy các nhóm cyclic xuất hiện đều có số phần tử chẵn), ta thấy độ dài bất biến của nhóm $(\mathbb{Z}/n)^\times$ chính là $f$ nếu $k=0,1$, là $f+1$ nếu $k=2$, và là $f+2$ nếu $k \ge 3$.

Kết luận: Nếu số ước nguyên tố phân biệt của $n$ là $f'$ thì số phần tử sinh nhỏ nhất của nhóm nhân khác số nguyên khả nghịch modulo $n$ là

  • $f'$ nếu $n$ lẻ hoặc $n \equiv 4 \pmod 8$,
  • $f'-1$ nếu $n \equiv 2 \pmod 4$,
  • $f'+1$ nếu $n$ chia hết cho $8$.



#742122 CMR: tồn tại duy nhất $(u_{1},..,u_{H})$ và...

Đã gửi bởi nmlinh16 on 13-11-2023 - 23:14 trong Số học

Đề này không biết bạn lấy từ đâu? Có rất nhiều lỗi sai trong phát biểu. Bạn phải phát biểu lại đúng thì mới làm được.

 

Đầu tiên lấy $n = 8$ thì $d = 4 = 2^2$, và lấy $a$ là số lẻ tùy ý. Từ đây thấy không có số nguyên lẻ $u$ nào để $\ord_8(u) = 4$ cả (hãy xem cả 4 lớp đồng dư khả nghịch trong $\mathbb{Z}/8$, bình phương của chúng đều bằng $\bar{1}$). Nói chung cần lấy $n$ là số nguyên dương không chia hết cho $8$ để đảm bảo rằng các cnhosm $\mathbb{Z}/p_i^{k_i}$ là các nhóm cyclic.

 

Tiếp theo là $(u_1,\ldots,u_H)$ và $(L_1,\ldots,L_H)$ duy nhất theo nghĩa như nào? Mình đồ rằng ý bạn là các số $u_t$ duy nhất modulo $n$. Và với các phát biểu kia thì mình đang hiểu là $a$ cho trước, và hai bộ $(u_1,\ldots,u_H)$ và $(L_1,\ldots,L_H)$ là duy nhất, phụ thuộc vào $a$. Nhưng cái này cũng không đúng: Lấy $n = 5$ thì $d = 4 = 2^2$ và lấy $a=1$. Khi đó có có thể lấy $u = 2$ và $L = 4$, hoặc $u' = 3$ và $L'=4$ để có được $\ord_5(2) = \ord_5(3) = 4$ và $u^L \equiv (u')^{L'} \equiv 1 \pmod 5$.




#742036 CM: nếu $A\in M_{3\times 2}(\mathbb{R...

Đã gửi bởi nmlinh16 on 05-11-2023 - 18:36 trong Đại số tuyến tính, Hình học giải tích

Ký hiệu $e_1 = (1,0,0), e_2 = (0,1,0), e_3 = (0,0,1)$. Thế thì 3 hàng của $A$ lần lượt là $e_1 A, e_2A, e_3A$. Chúng là 3 vectơ trong $\mathbb{R}^2$ nên phụ thuộc tuyến tính, tức là tồn tại $(\lambda_1,\lambda_2,\lambda_3) \in \mathbb{R}^3 \setminus \{(0,0,0)\}$ sao cho $$\lambda_1 e_1 A + \lambda_2 e_2 A + \lambda_3 e_3 A = 0,$$ suy ra $$\lambda_1 e_1 AA^T + \lambda_2 e_2 AA^T + \lambda_3 e_3 AA^T = 0,$$ tức là 3 hàng của $AA^T$ phụ thuộc tuyến tính, nên $\det(AA^T) = 0$.




#741986 Hàm khả vi

Đã gửi bởi nmlinh16 on 03-11-2023 - 00:22 trong Hàm số - Đạo hàm

https://math.berkele...weierstrass.pdf




#741963 Nên đọc sách gì cho môn Đại số đại cương

Đã gửi bởi nmlinh16 on 01-11-2023 - 21:18 trong Kinh nghiệm học toán

[Dummit, Foot] Abstract Algebra




#741935 Công Thức cực kỳ Đơn Giản để Tìm Đạo Hàm thông qua Hàm Ngược

Đã gửi bởi nmlinh16 on 30-10-2023 - 20:10 trong Tích phân - Nguyên hàm

?




#741906 $e^{A+B}=e^A\,e^B=e^B\,e^A$

Đã gửi bởi nmlinh16 on 29-10-2023 - 16:25 trong Đại số tuyến tính, Hình học giải tích

Vì $A$ và $B$ giao hoán nên ta có công thức khai triển nhị thức Newton (có thể chứng minh bằng quy nạp theo $k$): $$(A+B)^k = \sum_{i+j=k} \frac{k!}{i! j!} A^i B^j = \sum_{i+j = k} \frac{k!}{i! j!} B^j A^i,$$ hay $$\frac{1}{k!}(A+B)^k = \sum_{i+j=k} \frac{1}{i! j!} A^i B^j = \sum_{i+j = k} \frac{1}{i! j!} B^j A^i$$

Từ đó $$e^A e^B = \left(\sum_{i \ge 0} \frac{1}{i!} A^i\right) \left(\sum_{j \ge 0} \frac{1}{j!} B^j\right) = \sum_{i,j \ge 0} \frac{1}{i! j!} A^i B^j = \sum_{k \ge 0} \sum_{i+j=k} \frac{1}{i! j!} A^i B^j = \sum_{k \ge 0} \frac{1}{k!}(A+B)^k = e^{A+B}.$$ Tương tự, $$e^B e^A = e^{B+A} = e^{A+B}.$$

Đẳng thức trên vẫn đúng cho hai ma trận $A$ và $B$ nếu chúng giao hoán (nhưng không cần lũy linh) nếu các phần tử của hai ma trận này nằm trong một trường có giá trị tuyệt đối (điều này cho phép trang bị cho không gian các ma trận vuông một chuẩn, từ đó nói về khái niệm hội tụ của chuỗi). 




#741754 CMR nếu $30 | n_1^4+n_2^4 + \ldots + n_{31}^4$ thì g...

Đã gửi bởi nmlinh16 on 16-10-2023 - 17:43 trong Số học

Nhân xét rằng $n_4 > 5$ nên $n_i$ không chia hết cho $2, 3, 5$ với mọi $i \ge 4$. Theo định lý nhỏ Fermat thì $n_i^4 \equiv 1 \pmod 2$, $n_i^4 \equiv 1 \pmod 3$ và $n_i^4 \equiv 1 \pmod 5$, suy ra $n_i^4 \equiv 1 \pmod{30}$ với mọi $i \ge 4$. Từ đó $n_1^4 + n_2^4 + n_3^4 \equiv 2 \pmod{30}$.
Từ đây ta thấy $n_1, n_2, n_3$ không thể cùng lẻ, nên $n_1 = 2$, suy ra $n_2^4 + n_3^4 \equiv 1 \pmod{15}$.

Nếu $n_2 > 3 $ thì $n_2$ và $n_3$ đều không chia hết cho $3$, suy ra $n_2^4 \equiv n_3^4 \equiv 1 \pmod 3$, suy ra $n_2^4 + n_3^4 \equiv 2 \pmod 3$, mâu thuẫn. Vậy $n_2 = 3$, suy ra $n_3^4 \equiv 0 \pmod{5}$, từ đó $n_3 = 5$. Vậy $n_1, n_2, n_3$ là 3 số nguyên tố liên tiếp.




#741733 tìm cực trị hàm f(x,y)=$2x^{4} +y^{4}-x^{2...

Đã gửi bởi nmlinh16 on 15-10-2023 - 14:29 trong Giải tích

Cực đại và cực tiểu là khái niệm mang tính địa phương. Cực đại chỉ là giá trị lớn nhất của hàm trong lân cận của điểm đó.




#741552 đánh giá tích phân suy rộng sau là hội tụ hay phân kì $\int_{1...

Đã gửi bởi nmlinh16 on 28-09-2023 - 01:31 trong Giải tích

Với $x \ge 1$ thì $0 < \frac{\sqrt{x+1}}{x^2} \le \frac{\sqrt{x+x}}{x^2} = \frac{\sqrt{2}}{x\sqrt{x}}$. Mặt khác, tích phân $\int_1^{\infty} \frac{\sqrt{2}}{x\sqrt{x}}$ hội tụ. Thật vậy, $$\lim_{M \to \infty} \int_1^M \frac{\sqrt{2}}{x\sqrt{x}} = \lim_{M \to \infty} \left(-\frac{\sqrt{2}}{2\sqrt{x}} \Bigg|_{x = 1}^{x=M} \right) = \lim_{M \to \infty}\left(\frac{1}{\sqrt{2}} - \frac{1}{\sqrt{2M}}\right) = \frac{1}{\sqrt{2}}.$$ Theo tiêu chuẩn so sánh, tích phân đã cho hội tụ.




#741551 $\sin 2x + \sqrt 3 \cos 2x = 3m - 2$

Đã gửi bởi nmlinh16 on 28-09-2023 - 01:24 trong Công thức lượng giác, hàm số lượng giác

$-1$ là giá trị nhỏ nhất của hàm $\sin(t)$ khi $t$ chạy trên khoảng $\left(-\frac{2\pi}{3},\frac{\pi}{3}\right)$, đạt được khi $t = -\frac{\pi}{2}$.

Ngược lại ta luôn có $\sin(t) < \frac{\sqrt{3}}{2}$ với $t$ trên khoảng này, vì thế $\sin(t)$ luôn nằm trong $\left[-1, \frac{\sqrt{3}}{2}\right)$ như đã viết.

 

Bạn chú không mở topic mới cho cùng một câu hỏi. Nhờ mod xóa bài này.




#741010 Đại hội Toán học Việt Nam lần thứ X

Đã gửi bởi nmlinh16 on 12-08-2023 - 00:20 trong Tin tức - Vấn đề - Sự kiện

Bạn gái em, đang là sinh viên ngành toán.

Anh Hiệu vẫn đang ở khu Antony/Parc de Sceaux đó. Mùa xuân lúc hoa anh đào nở, ra đó, thì khả năng sẽ gặp được.

Em gái trong ảnh chắc là người yêu của @nmlinh16 đi ké hay sao, chứ dân làm Toán đã ít nữ rồi huống hồ lại còn xinh đẹp như vậy  :rolleyes:  
 
 

Cách đây ít năm thì anh Hiệu còn ở Sceaux (cạnh parc de sceaux), hàng xóm với một đứa em của Nesbit, bây giờ không biết đã chuyển đi chỗ khác chưa.




#741008 Dấu hiệu nhận biết hàm không tuần hoàn

Đã gửi bởi nmlinh16 on 12-08-2023 - 00:09 trong Công thức lượng giác, hàm số lượng giác

Cảm ơn ý kiến rất hay của bạn @tienmai. Khuyến khích mọi người chia sẻ thêm những kinh nghiệm tương tự.

Cảm ơn bạn đã trả lời và nêu suy nghĩ. Mình đồng tình với những suy nghĩ đó.
 
Tài liệu này rõ ràng hướng tới việc ôn tập thi trắc nghiệm (kì thi THPT quốc gia). Mình nghĩ là tài liệu này được nếu mục đích của người sử dụng là ôn thi THPT quốc gia.
 
Nhưng nếu bắt bẻ và soi xét thì vẫn có chỗ để nói. Chẳng hạn ở phần định nghĩa hàm tuần hoàn trong tài liệu có viết: "Số dương T nhỏ nhất thỏa mãn các tính chất trên được gọi là chu kì của hàm số tuần hoàn đó." Phát biểu chặt hơn là: "Nếu tồn tại số dương T nhỏ nhất thỏa mãn các tính chất trên thì số T được gọi là chu kì cơ sở của hàm số tuần hoàn đó." Tại sao lại vậy? Vì

  • Một hàm số có thể có nhiều chu kì, như hàm số $\sin$ có chu kì $2\pi, 4\pi, 6\pi\ldots$ còn chu kì cơ sở của hàm này là $2\pi$. 
  • Có những hàm tuần hoàn nhưng không có chu kì cơ sở. Một ví dụ cho điều này là hàm hằng số $c(x) = 0$ với miền xác định là tập số thực. Có các hàm khác tuần hoàn mà không có chu kì cơ sở, như là hàm Dirichlet (nếu học ngành Toán, ở môn giải tích thực, bạn sẽ gặp hàm này) nhưng hàm này sẽ không đời nào xuất hiện trong kì thi THPT quốc gia (nếu một ngày nó xuất hiện, toàn bộ ngành Toán trong nước sẽ dậy sóng).
Cũng như nhiều tài liệu nhằm ôn thi THPT quốc gia, tài liệu này tập trung vào hai thứ: các kết quả và bài tập, nhưng thiếu giải thích (rất nhiều). Mình đánh đồng các tài liệu này: chỉ liệt kê lại các kiến thức đã có ở sách giáo khoa cơ bản và nâng cao, chia thành từng phần rất nhỏ, từng miếng (mánh). Tài liệu khá tràn lan và tiểu tiết, thà rằng học từ gốc. Mình kể chuyện của bản thân: Khi còn học THPT, mình không học thêm Toán và chỉ dùng đến ba loại sách: sách giáo khoa cơ bản, nâng cao, sách chuyên toán (tài liệu chuyên toán đại số, giải tích, hình học 10, 11, 12). Chỉ như vậy mình đã thấy ổn áp với việc thi THPT quốc gia, và ít năm về sau có vài lần giúp người quen ôn thi.
 
Cuối cùng, việc bây giờ bạn mới thích toán không phải là muộn. À mà cũng còn phụ thuộc vào mục đích học Toán. Mình đến lúc còn hai năm là tốt nghiệp đại học mới bắt đầu tự học Toán qua sách bậc đại học. Nếu bạn có thắc mắc gì về định hướng với Toán thì có nhiều thành viên diễn đàn có thể giải đáp cho bạn (tất nhiên là ở một chủ đề khác).



#740976 Đại hội Toán học Việt Nam lần thứ X

Đã gửi bởi nmlinh16 on 09-08-2023 - 17:04 trong Tin tức - Vấn đề - Sự kiện

Báo cáo của em/mình ở Tiểu ban Hình học - Tôpô.

Hình gửi kèm

  • CA33202F-CC7F-4BF5-A712-106563364A8D.jpeg



#740970 Đại hội Toán học Việt Nam lần thứ X

Đã gửi bởi nmlinh16 on 09-08-2023 - 00:43 trong Tin tức - Vấn đề - Sự kiện

Gặp anh @RongChoi

0A7D35B3-A3B4-44BA-A54F-DBC23E9EC8AB.jpeg



#740950 Tìm hạt nhân, ảnh, hạng của ánh xạ tuyến tính

Đã gửi bởi nmlinh16 on 06-08-2023 - 19:05 trong Đại số tuyến tính, Hình học giải tích

Câu hỏi "tìm hạt nhân và ảnh" là một câu hỏi mơ hồ. Vì một không gian vectơ con của $\mathbb{R}^n$ có thể mô tả bởi các cách khác nhau: bởi một hệ sinh, bởi một cơ sở (một cách tương đương là bởi một hệ phương trình tham số), hoặc bởi một hệ phương trình tổng quát.




#740908 Chứng minh rằng $S_n=n$ với mọi $n\geq5.$

Đã gửi bởi nmlinh16 on 03-08-2023 - 23:51 trong Tổ hợp và rời rạc

Anh có thể nói kĩ về nhận xét được không ạ, nếu mỗi cặp vận động viên đều nằm trong một tam giác thì sẽ có cả $n$ vận động viên vô địch tương đối?

 

Nếu mỗi cặp vận động viên $\{a, b\}$ đều nằm trong một tam giác dạng ($a$ thắng $b$, $b$ thắng $c$, $c$ thắng $a$) hoặc ($b$ thắng $a$, $a$ thắng $c$, $c$ thắng $b$) thì $a$ viết được tên $b$ và $b$ cũng viết được tên $a$. Nếu điều này đúng với mọi cặp vận động viên thì mỗi vận động viên đều viết được tên của $n-1$ vận động viên còn lại, nghĩa là mọi vận động viên đều vô địch tương đối.

Ngược lại, giả sử một giải đấu có cả $n$ vận động viên vô địch tương đối. Xét một cặp vận động viên $\{a,b\}$ tùy ý. Ta có thể giả sử $a$ thắng $b$. Vì $b$ viết được tên $a$ tên tồn tại một vận động viên $c$ mà $b$ thắng $c$ và $c$ thắng $a$, vậy ta thu được tam giác $(a,b,c)$ chứa cặp $\{a,b\}$.